Given these four points: A(3, 3), B(−5, 7), C(2, 11), and D(9, −2), find the coordinates of the midpoint of line segments AB and CD.

Answers

Answer 1

Midpoint formula: (x1 + x2)/2 , (y1 + y2)/2

Midpoint AB = (3 +-5)/2, (3 + 7)/2 = -2/2 , 10/2 = (-1,5)

Midpoint CD = (2 +9)/2, (11 + -2)/2 = (11/2,9/2)


Related Questions

What is the error in this problem

Answers

Answer:

10). m∠x = 47°

11). x = 30.96

Step-by-step explanation:

10). By applying Sine rule in the given triangle DEF,

   [tex]\frac{\text{SinF}}{\text{DE}}=\frac{\text{SinD}}{\text{EF}}[/tex]

   [tex]\frac{\text{Sinx}}{7}=\frac{\text{Sin110}}{9}[/tex]

   Sin(x) = [tex]\frac{7\times (\text{Sin110})}{9}[/tex]

   Sin(x) = 0.7309

   m∠x = [tex]\text{Sin}^{-1}(0.7309)[/tex]

   m∠x = 46.96°

   m∠x ≈ 47°

11). By applying Sine rule in ΔRST,

   [tex]\frac{\text{SinR}}{\text{ST}}=\frac{\text{SinT}}{\text{RS}}[/tex]

   [tex]\frac{\text{Sin120}}{35}=\frac{\text{Sin50}}{x}[/tex]

   x = [tex]\frac{35\times (\text{Sin50})}{\text{Sin120}}[/tex]

   x = 30.96   

There are 30 colored marbles inside a bag. Six marbles are yellow, 9 are red, 7 are white, and 8 are blue. One is drawn at random. Which color is most likely to be chosen? A. white B. red C. blue D. yellow Include ALL work please!

Answers

Answer:

red

Step-by-step explanation:

Since the bag contains more red marbles than any other color, you are most likely to pick a red marble

Find the missing coordinate

Answers

Answer:

(0, -10a)

Step-by-step explanation:

From the picture attached,

Coordinates of a point have been given as (-10a, 0)

x-coordinate → distance of the point from the origin on x-axis

y-coordinate → distance of the point from the origin on y-axis

Therefore, distance of the given point on x-axis = -10a [(-) sign denotes the negative side of the x-axis]

Distance of the other point with unknown coordinates (x, y) (on y-axis) from the origin = y

And y = 10a

Therefore, coordinates of the unknown point will be (0, -10a).

[Here (-) sign denotes the negative side of the y-axis]

A laboratory tested n = 98 chicken eggs and found that the mean amount of cholesterol was LaTeX: \bar{x}x ¯ = 86 milligrams with σ = 7 milligrams. Find the margin of error E corresponding to a 95% confidence interval for the true mean cholesterol content, μ, of all such eggs.

Answers

Answer:

1.3859

Step-by-step explanation:

The formula for Margin of Error is given as:

Margin of Error = Critical value × Standard Error

Critical value = z score

In the question, we are given a confidence interval of 95%.

Z score for a 95% confidence level is given as: 1.96

Hence, critical value = 1.96

Standard Error = σ / √n

Where n = number of samples = 98 chicken eggs

σ = Standard deviation = 7 milligrams

Standard Error = 7/√98

Standard Error = 0.7071067812

Hence, Margin of Error = Critical value × Standard Error

Margin of Error = 1.96 × 0.7071067812

Margin of Error = 1.3859292911

Therefore, the margin of error corresponding to a 95% confidence interval for the true mean cholesterol content, μ, of all such eggs is approximately 1.3859

Question: The hypotenuse of a right triangle has a length of 14 units and a side that is 9 units long. Which equation can be used to find the length of the remaining side?

Answers

Answer:

The hypotenuse is the longest side in a triangle.

a^2=b^2+c^2.

14^2=9^2+c^2.

c^2=196-81.

c^2=115.

c=√115.

c=10.72~11cm

-8 + (-15)
Evaluate this expression ​

Answers

Answer:

-23

Step-by-step explanation:

-8+(-15) means that you are subtracting 15 from -8. So you end up with -8-15=-23.

HELP ASAP PLS :Find all the missing elements:

Answers

Answer:

a ≈ 1.59

b ≈ 6.69

Step-by-step explanation:

Law of Sines: [tex]\frac{a}{sinA} =\frac{b}{sinB} =\frac{c}{sinC}[/tex]

Step 1: Find c using Law of Sines

[tex]\frac{6}{sin58} =\frac{c}{sin13}[/tex]

[tex]c = sin13(\frac{6}{sin58})[/tex]

c = 1.59154

Step 2: Find a using Law of Sines

[tex]\frac{6}{sin58} =\frac{a}{sin109}[/tex]

[tex]a = sin109(\frac{6}{sin58} )[/tex]

a = 6.68961

A thin metal plate, located in the xy-plane, has temperature T(x, y) at the point (x, y). Sketch some level curves (isothermals) if the temperature function is given by

T(x, y)= 100/1+x^2+2y^2

Answers

Answer:

Step-by-step explanation:

Given that:

[tex]T(x,y) = \dfrac{100}{1+x^2+y^2}[/tex]

This implies that the level curves of a function(f) of two variables relates with the curves with equation f(x,y) = c

here c is the constant.

[tex]c = \dfrac{100}{1+x^2+2y^2} \ \ \--- (1)[/tex]

By cross multiply

[tex]c({1+x^2+2y^2}) = 100[/tex]

[tex]1+x^2+2y^2 = \dfrac{100}{c}[/tex]

[tex]x^2+2y^2 = \dfrac{100}{c} - 1 \ \ -- (2)[/tex]

From (2); let assume that the values of c > 0 likewise c < 100, then the interval can be expressed as 0 < c <100.

Now,

[tex]\dfrac{(x)^2}{\dfrac{100}{c}-1 } + \dfrac{(y)^2}{\dfrac{50}{c}-\dfrac{1}{2} }=1[/tex]

This is the equation for the  family of the eclipses centred at (0,0) is :

[tex]\dfrac{x^2}{a^2}+\dfrac{y^2}{b^2}=1[/tex]

[tex]a^2 = \dfrac{100}{c} -1 \ \ and \ \ b^2 = \dfrac{50}{c}- \dfrac{1}{2}[/tex]

Therefore; the level of the curves are all the eclipses with the major axis:

[tex]a = \sqrt{\dfrac{100 }{c}-1}[/tex]  and a minor axis [tex]b = \sqrt{\dfrac{50 }{c}-\dfrac{1}{2}}[/tex]  which satisfies the values for which 0< c < 100.

The sketch of the level curves can be see in the attached image below.

Which of the following graphs accurately displays a parabola with its directrix and focus?

Answers

Answer:

Hey there!

The first graph is the correct answer. A point on the parabola is equally far from the focus as it is to the directrix.

Let me know if this helps :)

The graph that  accurately displays a parabola with its directrix and focus is the first graph.

How do we make graph of a function?

Suppose the considered function whose graph is to be made is  f(x)

The values of 'x' (also called input variable, or independent variable) are usually plotted on horizontal axis, and the output values  f(x) are plotted on the vertical axis.

They are together plotted on the point  (x,y) = (x, f(x))

This is why we usually write the functions as:  y = f(x)

A point shown in the graphs on the parabola is equally far from the focus as it is to the directrix.

Therefore, The first graph is the correct answer.

Learn more about graphing functions here:

https://brainly.com/question/14455421

#SPJ2

Given a right triangle with a hypotenuse of 6 cm and a leg of 4cm, what is the measure of the other leg of the triangle rounded to the tenths?

Answers

Answer:

4.5 cm

Step-by-step explanation:

a^2+b^2=c^2

A represents the leg we already know, which has a length of 4 cm. C represents the hypotenuse with a length of 6 cm:

4^2+b^2=6^2, simplified: 16+b^2=36

subtract 16 from both sides:

b^2=20

now find the square root of both sides and that is the length of the other leg.

sqrt20= 4.4721, which can be rounded to 4.5

Answer:

4.5 cm

Step-by-step explanation:

Since this is a right triangle, we can use the Pythagorean Theorem.

[tex]a^2+b^2=c^2[/tex]

where a and b are the legs and c is the hypotenuse.

One leg is unknown and the other is 4 cm. The hypotenuse is 6 cm.

[tex]a=a\\b=4\\c=6[/tex]

Substitute the values into the theorem.

[tex]a^2+4^2=6^2[/tex]

Evaluate the exponents first.

4^2= 4*4= 16

[tex]a^2+16=6^2[/tex]

6^2=6*6=36

[tex]a^2+16=36[/tex]

We want to find a, therefore we must get a by itself.

16 is being added on to a^2. The inverse of addition is subtraction. Subtract 16 from both sides of the equation.

[tex]a^2+16-16=36-16\\\\a^2=36-16\\\\a^2=20[/tex]

a is being squared. The inverse of a square is a square root. Take the square root of both sides.

[tex]\sqrt{a^2}=\sqrt{20} \\\\a=\sqrt{20} \\\\a=4.47213595[/tex]

Round to the nearest tenth. The 7 in the hundredth place tells us to round the 4 in the tenth place to a 5.

[tex]a=4.5[/tex]

Add appropriate units. In this case, centimeters.

a= 4.5 cm

The length of the other leg is about 4.5 centimeters.

Express the product of z1 and z2 in standard form given that [tex]z_{1} = 6[cos(\frac{2\pi }{5}) + isin(\frac{2\pi }{5})][/tex] and [tex]z_{2} = 2\sqrt{2} [cos(\frac{-\pi }{2}) + isin(\frac{-\pi }{2})][/tex]

Answers

Answer:

Solution : 5.244 - 16.140i

Step-by-step explanation:

If we want to express the two as a product, we would have the following expression.

[tex]-6\left[\cos \left(\frac{2\pi }{5}\right)+i\sin \left(\frac{2\pi }{5}\right)\right]\cdot 2\sqrt{2}\left[\cos \left(\frac{-\pi }{2}\right)+i\sin \left(\frac{-\pi \:}{2}\right)\right][/tex]

Now we have two trivial identities that we can apply here,

( 1 ) cos(- π / 2) = 0,

( 2 ) sin(- π / 2) = - 1

Substituting them,

= [tex]-6\cdot \:2\sqrt{2}\left(0-i\right)\left(\cos \left(\frac{2\pi }{5}\right)+i\sin \left(\frac{2\pi }{5}\right)\right)[/tex]

= [tex]-12\sqrt{2}\sin \left(\frac{2\pi }{5}\right)+12\sqrt{2}\cos \left(\frac{2\pi }{5}\right)i[/tex]

Again we have another two identities we can apply,

( 1 ) sin(x) = cos(π / 2 - x )

( 2 ) cos(x) = sin(π / 2 - x )

[tex]\sin \left(\frac{2\pi }{5}\right)=\cos \left(\frac{\pi }{2}-\frac{2\pi }{5}\right) = \frac{\sqrt{2}\sqrt{5+\sqrt{5}}}{4}[/tex]

[tex]\cos \left(\frac{2\pi }{5}\right)=\sin \left(\frac{\pi }{2}-\frac{2\pi }{5}\right) = \frac{\sqrt{2}\sqrt{3-\sqrt{5}}}{4}[/tex]

Substitute,

[tex]-12\sqrt{2}(\frac{\sqrt{2}\sqrt{5+\sqrt{5}}}{4}) + 12\sqrt{2}(\frac{\sqrt{2}\sqrt{3-\sqrt{5}}}{4})[/tex]

= [tex]-6\sqrt{5+\sqrt{5}}+6\sqrt{3-\sqrt{5}} i[/tex]

= [tex]-16.13996 + 5.24419i[/tex]

= [tex]5.24419i - 16.13996[/tex]

As you can see option d is the correct answer. 5.24419 is rounded to 5.244, and 16.13996 is rounded to 16.14.

Which choice shows the product of 22 and 49 ?

Answers

Answer:

1078

Step-by-step explanation:

The product of 22 and 49 is 1078.

Answer:

1078 is the product

Step-by-step explanation:

solve the system with elimination 4x+3y=1 -3x-6y=3

Answers

Answer:

x = 1, y = -1

Step-by-step explanation:

If we have the two equations:

[tex]4x+3y=1[/tex] and [tex]-3x - 6y = 3[/tex], we can look at which variable will be easiest to eliminate.

[tex]y[/tex] looks like it might be easy to get rid of, we just have to multiply [tex]4x+3y=1[/tex]  by 2 and y is gone (as -6y + 6y = 0).

So let's multiply the equation [tex]4x+3y=1[/tex]  by 2.

[tex]2(4x + 3y = 1)\\8x + 6y = 2[/tex]

Now we can add these equations

[tex]8x + 6y = 2\\-3x-6y=3\\[/tex]

------------------------

[tex]5x = 5[/tex]

Dividing both sides by 5, we get [tex]x = 1[/tex].

Now we can substitute x into an equation to find y.

[tex]4(1) + 3y = 1\\4 + 3y = 1\\3y = -3\\y = -1[/tex]

Hope this helped!

(b) Suppose you want to study the length of time devoted to commercial breaks for two different types of television programs. Identify the types of programs you want to study (e.g., sitcoms, sports events, movies, news, children's programs, etc.). How large should the sample be for a specified margin of error

Answers

Answer:

The correct option is b.

Step-by-step explanation:

The complete question is:

Suppose you want to study the length of time devoted to commercial breaks for two different types of television programs. Identify the types of programs you want to study (e.g., sitcoms, sports events, movies, news, children's programs, etc.). How large should the sample be for a specified margin of error.

(a) It depends only on the specified margin of error.

(b) It depends on not only the specified margin of error, but also on the confidence level.

(c) It depends only on the confidence level.

Solution:

The (1 - α) % confidence interval for population mean is:

[tex]CI=\bar x\pm z_{\alpha/2}\times \frac{\sigma}{\sqrt{n}}[/tex]  

The margin of error for this interval is:

 [tex]MOE=z_{\alpha/2}\times \frac{\sigma}{\sqrt{n}}[/tex]

Then the sample size formula is:

[tex]n=[\frac{z_{\alpha/2}\times \sigma}{MOE}]^{2}[/tex]

The sample size is dependent upon the confidence level (1 - α) %, the standard deviation and the desired margin of error.

Thus, the correct option is b.

The size of the sample 'n' depends on not only the specified margin of error, but also on the confidence level.

Given :

Suppose you want to study the length of time devoted to commercial breaks for two different types of television programs.

The following steps can be used in order to determine the size of the sample be for a specified margin of error:

Step 1 - The formula of the confidence interval is given below:

[tex]\rm CI =\bar{x}+z_{\alpha /2}\times \dfrac{\sigma }{\sqrt{n} }[/tex]

Step 2 - Now, for this interval, the formula of margin of error is given below:

[tex]\rm MOE = z_{\alpha /2}\times \dfrac{\sigma}{\sqrt{n} }[/tex]

Step 3 - Solve the above expression for sample size 'n'.

[tex]\rm n = \left(\dfrac{z_{\alpha /2}\times \sigma}{MOE}\right)^2[/tex]

From the above steps, it can be concluded that the correct option is B) It depends on not only the specified margin of error, but also on the confidence level.

For more information, refer to the link given below:

https://brainly.com/question/13990500

Suppose that $2000 is invested at a rate of 2.6% , compounded semiannually. Assuming that no withdrawals are made, find the total amount after 10 years.

Answers

Answer:

$2,589.52

Step-by-step explanation:

[tex] A = P(1 + \dfrac{r}{n})^{nt} [/tex]

We start with the compound interest formula above, where

A = future value

P = principal amount invested

r = annual rate of interest written as a decimal

n = number of times interest is compound per year

t = number of years

For this problem, we have

P = 2000

r = 0.026

n = 2

t = 10,

and we find A.

[tex] A = $2000(1 + \dfrac{0.026}{2})^{2 \times 10} [/tex]

[tex] A = $2589.52 [/tex]

Compound interest formula:

Total = principal x ( 1 + interest rate/compound) ^ (compounds x years)

Total = 2000 x 1+ 0.026/2^20

Total = $2,589.52

how would you write six times the square of a number

Answers

6 to the power of whatever number you are going by

Answer:

[tex]\huge \boxed{6x^2 }[/tex]

Step-by-step explanation:

6 times a number squared.

Let the number be [tex]x[/tex].

6 is multiplied to [tex]x[/tex] squared.

[tex]6 \times x^2[/tex]

Please give me the answer ASAP The average of 5 numbers is 7. If one of the five numbers is removed, the average of the four remaining numbers is 6. What is the value of the number that was removed Show Your Work

Answers

Answer:

The removed number is 11.

Step-by-step explanation:

Given that the average of 5 numbers is 7. So you have to find the total values of 5 numbers :

[tex]let \: x = total \: values[/tex]

[tex] \frac{x}{5} = 7[/tex]

[tex]x = 7 \times 5[/tex]

[tex]x = 35[/tex]

Assuming that the total values of 5 numbers is 35. Next, we have to find the removed number :

[tex]let \: y = removed \: number[/tex]

[tex] \frac{35 - y}{4} = 6[/tex]

[tex]35 - y = 6 \times 4[/tex]

[tex]35 - y = 24[/tex]

[tex]35 - 24 = y[/tex]

[tex]y = 11[/tex]

Okay, let's slightly generalize this

Average of [tex]n[/tex] numbers is [tex]a[/tex]

and then [tex]r[/tex] numbers are removed, and you're asked to find the sum of these [tex]r[/tex] numbers.

Solution:

If average of [tex]n[/tex] numbers is [tex]a[/tex] then the sum of all these numbers is [tex]n\cdot a[/tex]

Now we remove [tex]r[/tex] numbers, so we're left with [tex](n-r)[/tex] numbers. and their. average will be [tex]{\text{sum of these } (n-r) \text{ numbers} \over (n-r)}[/tex] let's call this new average [tex] a^{\prime}[/tex]

For simplicity, say, sum of these [tex]r[/tex] numbers, which are removed is denoted by [tex]x[/tex] .

so the new average is [tex]\frac{\text{Sum of } n \text{ numbers} - x}{n-r}=a^{\prime}[/tex]

or, [tex] \frac{n\cdot a -x}{n-r}=a^{\prime}[/tex]

Simplify the equation, and solve for [tex]x[/tex] to get,

[tex] x= n\cdot a -a^{\prime}(n-r)=n(a-a^{\prime})+ra^{\prime}[/tex]

Hope you understand it :)

The angles of a quadrilateral are (3x + 2), (x-3), (2x+1), and 2(2x+5). Find x.

Answers

Answer:

3x+2+x-3+2x+1+2(2x+5)=360

10x+10=360

x=35

The top speed of this coaster is
128 mph. What is the tallest peak
of this coaster?
** Hint... convert mph into m/s.*​

Answers

To convert miles per hour to meters per second divide by 2.237

128 miles per hour / 2.237 = 57.22 meters per second.

Using the first equation:

57.22 = sqrt(2 x 9.81 x h)

Remove the sqrt by raising both sides to the second power:

57.22^2 = (2 x 9.81 x h)

Simplify Both sides:

3274.1284 = 19.62h

Divide both sides by 19.62:

H = 3274.1284/ 19.62

H = 166.88 meters


An apartment building is infested with 6.2 X 10 ratsOn average, each of these rats
produces 5.5 X 10' offspring each year. Assuming no rats leave or die, how many additional
rats will live in this building one year from now? Write your answer in standard form.

Answers

Answer: 3.41x10^3

Step-by-step explanation:

At the beginning of the year, we have:

R = 6.2x10 rats.

And we know that, in one year, each rat produces:

O = 5.5x10 offsprins.

Then each one of the 6.2x10 initial rats will produce 5.5x10 offsprings in one year, then after one year we have a total of:

(6.2x10)*(5.5x10) = (6.2*5.5)x(10*10) = 34.1x10^2

and we can write:

34.1 = 3.41x10

then: 34.1x10^2 = 3.41x10^3

So after one year, the average number of rats is:  3.41x10^3

I need help on this question :(​

Answers

Answer: 40 degree

Explanation:

FT bisect angle EFD dividing it into 2 equal angles (EFT and DFT)

And EFD = 80

We get :
EFT = 80/2
EFT = 40

And EFT + DFT = EFD = 80 degree

Therefore EFT = 40 degrees

The length of the sides of the triangle are in the ratio 3:4:5 and it’s perimeter is 144 cm find its area and height corresponding to the longest side

Answers

3:4:5 is one of Pythagorean’s triplets, meaning this is a right triangle.

144 = 3x + 4x + 5x (combine like terms)
144 = 12x (divide both sides by 12)
x = 12
To find the side lengths, multiply 3, 4 and 5 by 12. This means the triangle has sides of 36, 48 and 60 centimetres.

Because this is a right triangle, two of the sides are the base and the height. The hypotenuse (longest side) is not the base nor the height because it is not directly adjacent to the right angle. In this case, the hypotenuse is 60 cm.
A = bh/2
A = (36 cm)(48 cm)/2
A = 864 cm^2
Therefore the area is 864 cm^2.

I’m not sure what is meant by the “height corresponding to the longest side”, sorry.

Hope this helps!

what is the domain of f(x)=(1/4)^x

Answers

Answer:

B All real numbers

hope you wil understand

Answer:

[tex]\boxed{\sf B. \ All \ real \ numbers}[/tex]

Step-by-step explanation:

The domain is all possible values for x.

[tex]f(x)=(\frac{1}{4} )^x[/tex]

There are no restrictions on the value of x.

The domain is all real numbers.

The area of a rectangular garden if 6045 ft2. If the length of the garden is 93 feet, what is its width?

Answers

Answer:

65 ft

Step-by-step explanation:

The area of a rectangle is

A = lw

6045 = 93*w

Divide each side by 93

6045/93 = 93w/93

65 =w

Answer:

[tex]\huge \boxed{\mathrm{65 \ feet}}[/tex]

Step-by-step explanation:

The area of a rectangle formula is given as,

[tex]\mathrm{area = length \times width}[/tex]

The area and length are given.

[tex]6045=93 \times w[/tex]

Solve for w.

Divide both sides by 93.

[tex]65=w[/tex]

The width of the rectangular garden is 65 feet.

Let E and F be two events of an experiment with sample space S. Suppose P(E) = 0.6, P(F) = 0.3, and P(E ∩ F) = 0.1. Compute the values below.

(a) P(E ∪ F) =



(b) P(Ec) =



(c) P(Fc ) =



(d) P(Ec ∩ F) =

Answers

Answer:

(a) P(E∪F)= 0.8

(b) P(Ec)= 0.4

(c) P(Fc)= 0.7

(d) P(Ec∩F)= 0.8

Step-by-step explanation:

(a) It is called a union of two events A and B, and A ∪ B (read as "A union B") is designated to the event formed by all the elements of A and all of B. The event A∪B occurs when they do A or B or both.

If the events are not mutually exclusive, the union of A and B is the sum of the probabilities of the events together, from which the probability of the intersection of the events will be subtracted:

P(A∪B) = P(A) + P(B) - P(A∩B)

In this case:

P(E∪F)= P(E) + P(F) - P(E∩F)

Being P(E) = 0.6, P(F) = 0.3 and P(E ∩ F) = 0.1

P(E∪F)= 0.6 + 0.3 - 0.1

P(E∪F)= 0.8

(b)  The complement of an event A is defined as the set that contains all the elements of the sample space that do not belong to A.  The Complementary Rule establishes that the sum of the probabilities of an event and its complement must be equal to 1. So, if P (A) is the probability that an event A occurs, then the probability that A does NOT occur is  P (Ac) = 1- P (A)

In this case: P(Ec)= 1 - P(E)

Then: P(Ec)= 1 - 0.6

P(Ec)= 0.4

(c) In this case: P(Fc)= 1 - P(F)

Then: P(Fc)= 1 - 0.3

P(Fc)= 0.7

(d)  The intersection of two events A and B, designated as A ∩ B (read as "A intersection B") is the event formed by the elements that belong simultaneously to A and B. The event A ∩ B occurs when A and B do at once.

As mentioned, the complementary rule states that the sum of the probabilities of an event and its complement must equal 1. Then:

P(Ec intersection F) + P(E intersection F) = P(F)

P(Ec intersection F) + 0.1 = 0.3

P(Ec intersection F)= 0.2

Being:

P(Ec∪F)= P(Ec) + P(F) - P(Ec∩F)

you get:

P(Ec∩F)= P(Ec) + P(F) - P(Ec∪F)

So:

P(Ec∩F)= 0.4 + 0.3 - 0.2

P(Ec∩F)= 0.8

Identifying the Property of Equality

Quick

Check

Identify the correct property of equality to solve each equation.

3+x= 27

X/6 = 5

Answers

Answer:

a) Compatibility of Equality with Addition, b) Compatibility of Equality with Multiplication

Step-by-step explanation:

a) This expression can be solved by using the Compatibility of Equality with Addition, that is:

1) [tex]3+x = 27[/tex] Given

2) [tex]x+3 = 27[/tex] Commutative property

3) [tex](x + 3)+(-3) = 27 +(-3)[/tex] Compatibility of Equality with Addition

4) [tex]x + [3+(-3)] = 27+(-3)[/tex] Associative property

5) [tex]x + 0 = 27-3[/tex] Existence of Additive Inverse/Definition of subtraction

6) [tex]x=24[/tex] Modulative property/Subtraction/Result.

b) This expression can be solved by using the Compatibility of Equality with Multiplication, that is:

1) [tex]\frac{x}{6} = 5[/tex] Given

2) [tex](6)^{-1}\cdot x = 5[/tex] Definition of division

3) [tex]6\cdot [(6)^{-1}\cdot x] = 5 \cdot 6[/tex] Compatibility of Equality with Multiplication

4) [tex][6\cdot (6)^{-1}]\cdot x = 30[/tex] Associative property

5) [tex]1\cdot x = 30[/tex] Existence of multiplicative inverse

6) [tex]x = 30[/tex] Modulative property/Result

Answer:

3 + x = 27

✔ subtraction property of equality with 3

x over 6  = 5

✔ multiplication property of equality with 6

A political candidate has asked his/her assistant to conduct a poll to determine the percentage of people in the community that supports him/her. If the candidate wants a 10% margin of error at a 95% confidence level, what size of sample is needed

Answers

Answer:

The desired sample size is 97.

Step-by-step explanation:

Assume that 50% people in the community that supports the political candidate.

It is provided that the candidate wants a 10% margin of error (MOE) at a 95% confidence level.

The confidence interval for the population proportion is:

[tex]CI=\hat p\pm z_{\alpha/2}\sqrt{\frac{\hat p(1-\hat p)}{n}}[/tex]

Then the margin of error is:

[tex]MOE= z_{\alpha/2}\sqrt{\frac{\hat p(1-\hat p)}{n}}[/tex]

Compute the critical value of z as follows:

[tex]z_{\alpha/2}=z_{0.05/2}=z_{0.025}=1.96[/tex]

*Use a z-table.

Compute the sample size as follows:

[tex]MOE= z_{\alpha/2}\sqrt{\frac{\hat p(1-\hat p)}{n}}[/tex]

       [tex]n=[\frac{z_{\alpha/2}\times \sqrt{\hat p(1-\hat p)} }{MOE}]^{2}[/tex]

          [tex]=[\frac{1.96\times \sqrt{0.50(1-0.50)} }{0.10}^{2}\\\\=[9.8]^{2}\\\\=96.04\\\\\approx 97[/tex]

Thus, the desired sample size is 97.

Find the value of the expression: −mb −m^2 for m=3.48 and b=96.52

Answers

Answer:

The value of the expression when [tex]m = 3.48[/tex] and [tex]b = 96.52[/tex] is 323.779.

Step-by-step explanation:

Let be [tex]f(m, b) = m\cdot b - m^{2}[/tex], if [tex]m = 3.48[/tex] and [tex]b = 96.52[/tex], the value of the expression:

[tex]f(3.48,96.52) = (3.48)\cdot (96.52)-3.48^{2}[/tex]

[tex]f(3.48,96.52) = 323.779[/tex]

The value of the expression when [tex]m = 3.48[/tex] and [tex]b = 96.52[/tex] is 323.779.

given point (-6, -3) and a slope of 4, write an equation in point-slope form

Answers

Answer:

y = 4x + 21

Step-by-step explanation:

Hello!

Point-slope form is y - y1 = m(x - x1)

y1 is the y point

x1 is the x point

m is the slope

Put in what you know

y - -3 = 4(x - -6)

Subtracting a negative is the same as adding

y + 3 = 4(x + 6)

Distribute the 4

y + 3 = 4x + 24

Subtract 3 from both sides

y = 4x + 21

The answer is y = 4x + 21

Hope this helps!

can anyone show me this in verbal form?

Answers

Answer:

2 * (x + 2) = 50

Step-by-step explanation:

Let's call the unknown number x. "A number and 2" means that we need to add the numbers, therefore it would be x + 2. "Twice" means 2 times a quantity so "twice a number and 2" would be 2 * (x + 2). "Is" denotes that we need to use the "=" sign and because 50 comes after "is", we know that 50 goes on the right side of the "=" so the final answer is 2 * (x + 2) = 50.

Other Questions
i need help. Can u help me solve for x? Which of the following defines what privacy policy is? A. A legal document that an app or website must provide which describes what user information they collect and how they use it. B. A legal document that an app or website must provide which describes the rules that the company and users must obey when they use the app C. Choices a website or app provides users about what information is collected and visible to others FORM A CUBIC POLYNOMIAL WHOSE ZEROS ARE -3,-1 AND 2 Find two positive fractions that subtract to 1/6. Write your answer as fraction -fraction please help me to answer this question write the fraction for each of the following do number 3 and 4 thanks where in india does the heavy monsoon pre-monsoon occur A setback of affirmative action is that: a. those benefitting from affirmative action begin to experience self-doubts about their competence and merit. b. women and minorities usually feel deprived. c. employees start to overpower the management. d. people who are the subject of affirmative action are viewed as being more qualified than they actually are. A person who obeys the rules because of a sense of social obligation to behave in certain ways is functioning at which level of moral development? I need help on this question The world on a turtles back Answer the following questions properly (complete sentences and direct citations/quotations where necessary) 6. Explain a specific symbol in the story, or what a larger part of the story symbolizes. Help plz! Jim is climbing a mountain that has a base 150 feet above sea level. If he climbs 233 feet then descends into a cave 64 feet, how far above sea level is Jim Select the word from the list that best fits the definition ore from which aluminium is made Stem-Changing Verbs Spanish Mark each of the following as true or false and explain how you know. Webster Corporation's monthly projected general and administrative expenses include $5,600 administrative salaries, $3,000 of other cash administrative expenses, $1,650 of depreciation expense on the administrative equipment, and .5% monthly interest on an outstanding bank loan of $16,000. Compute the total general and administrative expenses to be reported on the general and administrative expense budget per month. Determine the number of solutions for each quadratic.1.) 4x^2-3x+4=02.)9x^2-12x+4=0PLEASE HELP!!! _____ media are specifically designed to help bring customers eyeball to eyeball with the product--often at the point of sale or close to it. Explain what personal involvement in leadership means. Why is being personally involved important? What are some ways a leader can get personally involved and show that he or she cares? What is the oxidizing agent in the redox reaction represented by the following cell notation? Ni(s) | Ni2+(aq) || Ag+(aq) | Ag(s)